K
Khách

Hãy nhập câu hỏi của bạn vào đây, nếu là tài khoản VIP, bạn sẽ được ưu tiên trả lời.

31 tháng 1 2018

Tìm được: Q = 2 a a − 3

28 tháng 6 2017

Phép trừ các phân thức đại số

28 tháng 8 2017

Nhìn là biết chụp hình lại ở sách hướng dẫn nào đó rồi

5 tháng 10 2019

a, ĐKXĐ: \(\hept{\begin{cases}x^3+1\ne0\\x^9+x^7-3x^2-3\ne0\\x^2+1\ne0\end{cases}}\)

b, \(Q=\left[\left(x^4-x+\frac{x-3}{x^3+1}\right).\frac{\left(x^3-2x^2+2x-1\right)\left(x+1\right)}{x^9+x^7-3x^2-3}+1-\frac{2\left(x+6\right)}{x^2+1}\right]\)

\(Q=\left[\frac{\left(x^3+1\right)\left(x^4-x\right)+x-3}{\left(x+1\right)\left(x^2-x+1\right)}.\frac{\left(x-1\right)\left(x+1\right)\left(x^2-x+1\right)}{\left(x^7-3\right)\left(x^2+1\right)}+1-\frac{2\left(x+6\right)}{x^2+1}\right]\)

\(Q=\left[\left(x^7-3\right).\frac{\left(x-1\right)}{\left(x^7-3\right)\left(x^2+1\right)}+1-\frac{2\left(x+6\right)}{x^2+1}\right]\)

\(Q=\frac{x-1+x^2+1-2x-12}{x^2+1}\)

\(Q=\frac{\left(x-4\right)\left(x+3\right)}{x^2+1}\)

28 tháng 1 2017

9/48; 10/48; 11/48; 12/48; 13/48; 14/48; 15/48

28 tháng 1 2017

\(\frac{1}{6}=\frac{8}{48}\)

\(\frac{1}{3}=\frac{12}{48}\)

Vậy \(\frac{8}{48}< \frac{9}{48};\frac{10}{48};\frac{11}{48}< \frac{12}{48}\)

Vậy a bằng 9;10;11

25 tháng 10 2020

Bài 4: Áp dụng bất đẳng thức AM - GM, ta có: \(P=\text{​​}\Sigma_{cyc}a\sqrt{b^3+1}=\Sigma_{cyc}a\sqrt{\left(b+1\right)\left(b^2-b+1\right)}\le\Sigma_{cyc}a.\frac{\left(b+1\right)+\left(b^2-b+1\right)}{2}=\Sigma_{cyc}\frac{ab^2+2a}{2}=\frac{1}{2}\left(ab^2+bc^2+ca^2\right)+3\)Giả sử b là số nằm giữa a và c thì \(\left(b-a\right)\left(b-c\right)\le0\Rightarrow b^2+ac\le ab+bc\)\(\Leftrightarrow ab^2+bc^2+ca^2\le a^2b+abc+bc^2\le a^2b+2abc+bc^2=b\left(a+c\right)^2=b\left(3-b\right)^2\)

Ta sẽ chứng minh: \(b\left(3-b\right)^2\le4\)(*)

Thật vậy: (*)\(\Leftrightarrow\left(b-4\right)\left(b-1\right)^2\le0\)(đúng với mọi \(b\in[0;3]\))

Từ đó suy ra \(\frac{1}{2}\left(ab^2+bc^2+ca^2\right)+3\le\frac{1}{2}.4+3=5\)

Đẳng thức xảy ra khi a = 2; b = 1; c = 0 và các hoán vị

26 tháng 10 2020

Bài 1: Đặt \(a=xc,b=yc\left(x,y>0\right)\)thì điều kiện giả thiết trở thành \(\left(x+1\right)\left(y+1\right)=4\)

Khi đó  \(P=\frac{x}{y+3}+\frac{y}{x+3}+\frac{xy}{x+y}=\frac{x^2+y^2+3\left(x+y\right)}{xy+3\left(x+y\right)+9}+\frac{xy}{x+y}\)\(=\frac{\left(x+y\right)^2+3\left(x+y\right)-2xy}{xy+3\left(x+y\right)+9}+\frac{xy}{x+y}\)

Có: \(\left(x+1\right)\left(y+1\right)=4\Rightarrow xy=3-\left(x+y\right)\)

Đặt \(t=x+y\left(0< t< 3\right)\Rightarrow xy=3-t\le\frac{\left(x+y\right)^2}{4}=\frac{t^2}{4}\Rightarrow t\ge2\)(do t > 0)

Lúc đó \(P=\frac{t^2+3t-2\left(3-t\right)}{3-t+3t+9}+\frac{3-t}{t}=\frac{t}{2}+\frac{3}{t}-\frac{3}{2}\ge2\sqrt{\frac{t}{2}.\frac{3}{t}}-\frac{3}{2}=\sqrt{6}-\frac{3}{2}\)với \(2\le t< 3\)

Vậy \(MinP=\sqrt{6}-\frac{3}{2}\)đạt được khi \(t=\sqrt{6}\)hay (x; y) là nghiệm của hệ \(\hept{\begin{cases}x+y=\sqrt{6}\\xy=3-\sqrt{6}\end{cases}}\)

Ta lại có \(P=\frac{t^2-3t+6}{2t}=\frac{\left(t-2\right)\left(t-3\right)}{2t}+1\le1\)(do \(2\le t< 3\))

Vậy \(MaxP=1\)đạt được khi t = 2 hay x = y = 1

1)

Xét \(\left|x\right|>3\)\(\Rightarrow\)\(C>0\)

Xét \(0\le\left|x\right|< 3\)\(\Rightarrow\)\(C< 0\)

+ Với \(\left|x\right|=0\)\(\Leftrightarrow\)\(x=0\) thì \(C=-2\)

+ Với \(\left|x\right|=1\)\(\Leftrightarrow\)\(x=\pm1\) thì \(C=-3\)

+ Với \(\left|x\right|=2\)\(\Leftrightarrow\)\(x=\pm2\) thì \(C=-6\)

Vậy GTNN của \(C=-6\) khi \(x=\pm2\)

2) 

Xét \(x\ge0\)\(\Rightarrow\)\(x-\left|x\right|=0\)

Xét \(x< 0\)\(\Rightarrow\)\(x-\left|x\right|=2x< 0\)

Vậy GTLN của \(x-\left|x\right|=0\) khi \(x>0\)

5 tháng 1 2020

Ví dụ một bài toán : 

Tìm GTLN của B = 10-4 | x-2| 

Vì |x-2| \(\ge0\forall x\)

\(\Rightarrow-4.\left|x-2\right|\le0\forall x\). Tại sao mà tìm GTLN mà lại nhỏ hơn hoặc bằng 0 ạ

23 tháng 12 2018

Ad bđt : \(xy+yz+zx\le x^2+y^2+z^2\) (Cái bđt này c/m dễ : Nhân 2 vế với 2 -> chuyển vế -> tổng bình phương > 0 luôn đúng)

Kết hợp với bđt Cô-si cho 2 số dương ta đc

\(\frac{a^3}{b}+\frac{b^3}{c}+\frac{c^3}{a}=\left(\frac{a^3}{b}+ab\right)+\left(\frac{b^3}{c}+bc\right)+\left(\frac{c^3}{a}+ac\right)-\left(ab+bc+ca\right)\)

                                   \(\ge2\sqrt{\frac{a^3}{b}.ab}+2\sqrt{\frac{b^3}{c}.bc}+2\sqrt{\frac{c^3}{a}.ac}-\left(a^2+b^2+c^2\right)\)

                                       \(=2a^2+2b^2+2c^2-a^2-b^2-c^2\)

                                        \(=a^2+b^2+c^2\)

\(\Rightarrow\frac{a^3}{b}+\frac{b^3}{c}+\frac{c^3}{a}\ge a^2+b^2+c^2\left(1\right)\)

Áp dụng bđt Cô-si cho 2 số dương

\(a^2+b^2\ge2ab\)

\(b^2+c^2\ge2bc\)

\(c^2+a^2\ge2ac\)

\(a^2+1\ge2a\)

\(b^2+1\ge2b\)

\(c^2+1\ge2c\)

Cộng từng vế của 6 bđt trên lại ta đc

\(3\left(a^2+b^2+c^2+1\right)\ge2\left(ab+bc+ca+a+b+c\right)\)

 \(\Leftrightarrow3\left(a^2+b^2+c^2+1\right)\ge2.6\)

\(\Leftrightarrow a^2+b^2+c^2+1\ge4\)

\(\Leftrightarrow a^2+b^2+c^2\ge3\left(2\right)\)

Từ (1) và (2) \(\Rightarrow\frac{a^3}{b}+\frac{b^3}{c}+\frac{c^3}{a}\ge a^2+b^2+c^2\ge3\)

Dấu "=" xảy ra \(\Leftrightarrow\hept{\begin{cases}a=b=c\\a+b+c+ab+bc+ca=6\end{cases}}\)

                         \(\Leftrightarrow\hept{\begin{cases}a=b=c\\a+a+a+aa+aa+aa=6\end{cases}}\)(thay hết b , c thành a)

                         \(\Leftrightarrow\hept{\begin{cases}a=b=c\\3a^2+3a=6\end{cases}}\)

                        \(\Leftrightarrow\hept{\begin{cases}a=b=c\\a^2+a-2=0\end{cases}}\)

                         \(\Leftrightarrow\hept{\begin{cases}a=b=c\\\left(a-1\right)\left(a+2\right)=0\end{cases}}\)

                          \(\Leftrightarrow a=b=c=1\)hoặc \(a=b=c=-2\)

Mà a,b,c là các số dương nên a = b = c  = 1

Vậy ............